A difficult logarithmic integral and its relation to alternating Euler Sums

closed-formeuler-sumsharmonic-numbersintegrationsequences-and-series

The following integral was recently brought up in this thread on AoPS.

$$\mathfrak I~=~\int_0^1\frac{\log(1-x)\log^2(x)\log(1+x)}{1+x}\mathrm dx\tag1$$

It is reasonable to ask for a closed-form of $(1)$ as similar (namely taking $x$ instead of $1+x$ as numerator) have known closed-form representations. The crux here appears to be the inherent alternating structure induced by both, the $1+x$ within the numerator as well as in the logarithm. Let me elaborate on this by converting this integral into a sum. Using the generating function for the harmonic numbers combined with various well-known results we may obtain

$$\small\begin{align*}
\int_0^1\frac{\log(1-x)\log^2(x)\log(1+x)}{1+x}\mathrm dx&=\sum_{n\geqslant1}(-1)^{n+1}H_n\int_0^1x^n\log^2(x)\log(1-x)\mathrm dx\\
&=\sum_{n\geqslant1}(-1)^{n+1}H_n\left(\frac{\mathrm d^2}{\mathrm dn^2}\left[-\frac{\psi^{(0)}(n+2)+\gamma}{n+1}\right]\right)\\
&=\sum_{n\geqslant1}(-1)^nH_n\left(2\frac{\psi^{(0)}(n+2)+\gamma}{(n+1)^3}-2\frac{\psi^{(1)}(n+2)}{(n+1)^2}+\frac{\psi^{(2)}(n+2)}{n+1}\right)\\
&=2\sum_{n\geqslant1}(-1)^nH_n\left(\frac{H_{n+1}}{(n+1)^3}-\frac{\zeta(2)-H_{n+1}^{(2)}}{(n+1)^2}-\frac{\zeta(3)-H_{n+1}^{(3)}}{n+1}\right)\\
&=2\sum_{n\geqslant1}(-1)^{n+1}\left(H_n-\frac1n\right)\left(\frac{H_n}{n^3}-\frac{\zeta(2)-H_n^{(2)}}{n^2}-\frac{\zeta(3)-H_n^{(3)}}n\right)
\end{align*}$$

So, basically we are left with alternating sums of the form $\sum\limits_{n\geqslant1}(-1)^n a_n$ where $a_n$ is a coefficient up to a weight of $5$ (by the usual definition of weight). Expanding the parenthesis (!) we are left with the following (ordered by weight and complexity)

$$\small\frac12\mathfrak I-\frac54\zeta(2)\zeta(3)=\zeta(3)\sum_{n\geqslant1}(-1)^n\frac{H_n}n+\zeta(2)\sum_{n\geqslant1}(-1)^n\frac{H_n}{n^2}+\sum_{n\geqslant1}(-1)^n\frac{H_n}{n^4}-\sum_{n\geqslant1}(-1)^n\frac{H_n^2}{n^3}\\\small-\sum_{n\geqslant1}(-1)^n\frac{H_nH_n^{(2)}}{n^2}-\sum_{n\geqslant1}(-1)^n\frac{H_nH_n^{(3)}}n+\sum_{n\geqslant1}(-1)^n\frac{H_n^{(2)}}{n^3}+\sum_{n\geqslant1}(-1)^n\frac{H_n^{(3)}}{n^2}$$

I am not entirely sure about splitting the sum as the first series is only conditionally convergent instead of absolutely convergent as the rest. However, the first two series fall rather easily by using the generating function again and integrating once and twice, respectively, giving us the following results.

\begin{align*}
\sum_{n\geqslant1}(-1)^n\frac{H_n}n&=\frac12\zeta(2)-\frac12\log^2(2)\tag2\\
\sum_{n\geqslant1}(-1)^n\frac{H_n}{n^2}&=-\frac58\zeta(3)\tag3
\end{align*}

While this approach essentially works for the third sum too the calculations are nearly impossible by hand and WolframAlpha already returns this monstrosity for a denominator of only $n^3$. But, let's put this aside as "in-fact-doable" (even though the result may not admit a closed-form in terms of known constants alone).

For the remaining series – except the sixth – I have a vague idea. Making use of more generating functions, namely the following, one can get these sums. Anyway, I do not know if the so-occurring integrals are easier than $(1)$ or in the worst case not even harder.

$$\small\begin{align*}
\sum_{n\geqslant1}H_n^{(p)}x^n&=\frac{\operatorname{Li}_p(x)}{1-x}\tag4\\
\sum_{n\geqslant1}H_n^2x^n&=\frac1{1-x}(\log^2(1-x)+\operatorname{Li}_2(x))\tag5\\
\sum_{n\geqslant1}H_nH_n^{(2)}x^n&=\frac1{1-x}\left(\frac12\log(x)\log^2(1-x)+\operatorname{Li}_3(x)+\operatorname{Li}_3(1-x)\tag6\\
-\zeta(2)\log(1-x)-\zeta(3)\right)
\end{align*}$$

However, there is no similar formula I am aware of to generate $H_nH_n^{(3)}$, so this part remains unknown. It might be possible to shorten things up by considering linear combinations of some of the occuring series but honestly speaking I am not longer able to keep track of all what is going on.

I have a few questions, but I would be glad for answers referring to only one of them too.

$\textbf{Q. 1}$ What is the current state of art concerning regular alternating Euler Sums? Is there a similar formula available as there is for non-alternating (I recall to have seen a post dealing with this issue on MSE, but I am unable to find it again)?

$\textbf{Q. 2}$ Are the given generating functions $(4)$$(6)$ of any use? To put it in other words: are the occurring integrals easier to handle than $(1)$ (e.g. avoiding alternating sums at all)? I played a little bit with them but soon I ran into problems concerning convergence and I was not able to resolve them.

$\textbf{Q. 3}$ How can we deal with $H_nH_n^{(3)}$ at all? Is there a generating function known for solely this coefficient or is it necessary to use harmonic series containing this coefficient among other (actually I know some of them)?

$\textbf{Q. 4}$ Is there a closed-form for $(1)$, possibly including non-expressible constants immanent to the field of polylogarithms?

Thanks in advance!


EDIT: As pointed out by user97357329 the series having the $H_nH_n^{(3)}$ can be found in Cornel I. Valean's (Almost) Impossible Integrals, Sums, and Series, derived on page $528-529$. Searching through the book I found all of the remaining series presented as problems $4.53,$ $4.54$, $4.55$ and $4.57$ (Thanks to Ali Shather, who noticed a crucial typo).

Best Answer

Different approach to compute our main sum $\displaystyle\sum_{n=1}^\infty(-1)^n\frac{H_nH_n^{(3)}}{n}$.


From here we have

$$\int_0^1\frac{\ln^ax\ln\left(\frac{1+x}{2}\right)}{1-x}=(-1)^aa!\sum_{n=1}^\infty\frac{(-1)^nH_n^{a+1}}{n}\tag{1}$$ Using the identity

$$\ln^2(1+x)=2\sum_{n=1}^\infty\frac{H_n}{n+1}(-x)^{n+1}=2\sum_{n=1}^\infty(-1)^n\left(\frac{H_n}{n}-\frac{1}{n^2}\right)x^n\tag{2}$$

Multiply both sides of (2) by $\frac{\ln^2x}{1-x}$ then integrate from $x=0$ to $1$ we have

\begin{align} I&=\int_0^1\frac{\ln^2x\ln^2(1+x)}{1-x}\ dx=2\sum_{n=1}^\infty(-1)^n\left(\frac{H_n}{n}-\frac{1}{n^2}\right)\int_0^1\frac{x^n\ln^2x}{1-x}\ dx\\ &=2\sum_{n=1}^\infty(-1)^n\left(\frac{H_n}{n}-\frac{1}{n^2}\right)\left(2\zeta(3)-2H_n^{(3)}\right)\\ &=4\zeta(3)\underbrace{\sum_{n=1}^\infty(-1)^n\left(\frac{H_n}{n}-\frac{1}{n^2}\right)}_{\text{use (2) where}\ x=1}+4\sum_{n=1}^\infty(-1)^n\frac{H_n^{(3)}}{n^2}-4\sum_{n=1}^\infty(-1)^n\frac{H_nH_n^{(3)}}{n}\\ &=2\ln^22\zeta(3)+4\sum_{n=1}^\infty(-1)^n\frac{H_n^{(3)}}{n^2}-4\sum_{n=1}^\infty(-1)^n\frac{H_nH_n^{(3)}}{n}\tag{3} \end{align}


On the other hand:

\begin{align} I&=\small{\int_0^1\frac{\ln^2x\ln^2(1+x)}{1-x}\ dx\overset{x\mapsto 1-x}=\int_0^1\frac{\ln^2(1-x)\ln^2(2-x)}{x}=\int_0^1\frac{\ln^2(1-x)}{x}\left(\ln2+\ln\left(1-\frac x2\right)\right)^2\ dx}\\ &=\small{\ln^22\int_0^1\frac{\ln^2(1-x)}{x}\ dx+2\ln2\underbrace{\int_0^1\frac{\ln^2(1-x)}{x}\ln\left(1-\frac x2\right)\ dx}_{x\mapsto 1-x}+\underbrace{\int_0^1\frac{\ln^2(1-x)}{x}\ln^2\left(1-\frac x2\right)\ dx}_{\text{use (2)}}}\\ &=\small{2\ln^22\zeta(3)+2\ln2\underbrace{\int_0^1\frac{\ln^2x}{1-x}\ln\left(\frac{1+x}{2}\right)\ dx}_{\text{use (1)}}+2\sum_{n=1}^\infty\frac1{2^n}\left(\frac{H_n}{n}-\frac1{n^2}\right)\int_0^1x^{n-1}\ln^2(1-x)\ dx}\\ &=2\ln^22\zeta(3)+4\ln2\sum_{n=1}^\infty(-1)^n\frac{H_n^{(3)}}{n}+2\sum_{n=1}^\infty\frac1{2^n}\left(\frac{H_n}{n}-\frac1{n^2}\right)\left(\frac{H_n^2+H_n^{(2)}}{n}\right)\\ &=\small{2\ln^22\zeta(3)+4\ln2\sum_{n=1}^\infty(-1)^n\frac{H_n^{(3)}}{n}+2\sum_{n=1}^\infty\frac{H_n^3}{n^22^n}+2\sum_{n=1}^\infty\frac{H_nH_n^{(2)}}{n^22^n}-2\sum_{n=1}^\infty\frac{H_n^2}{n^32^n}-2\sum_{n=1}^\infty\frac{H_n^{(2)}}{n^32^n}}\quad \quad \quad \quad \text{(4)} \end{align}

From (3) and (4) we conclude that

$$\sum_{n=1}^\infty(-1)^n\frac{H_nH_n^{(3)}}{n}=\\ \small{\sum_{n=1}^\infty(-1)^n\frac{H_n^{(3)}}{n^2}-\ln2\sum_{n=1}^\infty(-1)^n\frac{H_n^{(3)}}{n}-\frac12\sum_{n=1}^\infty\frac{H_n^3}{n^22^n}-\frac12\sum_{n=1}^\infty\frac{H_nH_n^{(2)}}{n^22^n} +\frac12\sum_{n=1}^\infty\frac{H_n^{(2)}}{n^32^n}+\frac12\sum_{n=1}^\infty\frac{H_n^2}{n^32^n}}\tag{5}$$


We have the following results:

$$S_1=\sum_{n=1}^\infty(-1)^n\frac{H_n^{(3)}}{n^2}=\frac{21}{32}\zeta(5)-\frac34\zeta(2)\zeta(3)$$

$$S_2=\sum_{n=1}^\infty(-1)^n\frac{H_n^{(3)}}{n}=\frac34\ln2\zeta(3)-\frac{19}{16}\zeta(4)$$

$$S_3=\sum_{n=1}^\infty\frac{H_n^3}{n^22^n}=-14\operatorname{Li}_5\left(\frac12\right)-9\ln2\operatorname{Li}_4\left(\frac12\right)+\frac{279}{16}\zeta(5)-\frac{25}{4}\ln2\zeta(4)-\frac78\zeta(2)\zeta(3)\\-\frac74\ln^22\zeta(3)+\frac{13}{12}\ln^32\zeta(2)-\frac{31}{120}\ln^52$$

$$S_4=\sum_{n=1}^\infty\frac{H_nH_n^{(2)}}{n^22^n}=2\operatorname{Li}_5\left(\frac12\right)+\ln2\operatorname{Li}_4\left(\frac12\right)-\frac{31}{32}\zeta(5)+\frac{1}{8}\ln2\zeta(4)+\frac18\zeta(2)\zeta(3)\\-\frac{1}{12}\ln^32\zeta(2)+\frac{1}{40}\ln^52$$

$$S_5=\sum_{n=1}^\infty\frac{H_n^{(2)}}{n^32^n}=-2\operatorname{Li}_5\left(\frac12\right)-3\ln2\operatorname{Li}_4\left(\frac12\right)+\frac{23}{64}\zeta(5)-\frac1{16}\ln2\zeta(4)+\frac{23}{16}\zeta(2)\zeta(3)\\-\frac{23}{16}\ln^22\zeta(3)+\frac7{12}\ln^32\zeta(2)-\frac{13}{120}\ln^52$$

$$S_6=\sum_{n=1}^\infty\frac{H_n^2}{n^32^n}=-2\operatorname{Li}_5\left(\frac12\right)-\ln2\operatorname{Li}_4\left(\frac12\right)+\frac{279}{64}\zeta(5)-\frac{37}{16}\ln2\zeta(4)-\frac{9}{16}\zeta(2)\zeta(3)\\+\frac{7}{16}\ln^22\zeta(3)+\frac1{12}\ln^32\zeta(2)-\frac{1}{40}\ln^52$$


By substituting these results in (5) we get

$$\sum_{n=1}^\infty(-1)^n\frac{H_nH_n^{(3)}}{n}=4 \operatorname{Li}_5\left(\frac{1}{2}\right)+2\ln2\operatorname{Li}_4\left(\frac{1}{2}\right)-\frac{167}{32}\zeta(5)+\frac{49}{16}\ln2\zeta(4)-\frac{3}{8}\ln^22\zeta(3)\\-\frac{1}{6}\ln^32\zeta(2)+\frac{1}{16}\zeta(2)\zeta(3)+\frac{1}{20}\ln^52$$


NOTE:

$S_1$ and $S_2$ can be found here, $S_3$ and $S_4$ can be found here and $S_5$ and $S_6$ can be found here.